Rational numbers and Lowest terms proof:

Click For Summary
SUMMARY

The discussion focuses on proving properties of rational numbers in lowest terms. Specifically, it addresses two main problems: proving that if \(\frac{r}{s} = \frac{t}{u}\) with both fractions in lowest terms, then \(r=t\) and \(s=u\); and proving that an integer \(N\) cannot equal \(\frac{r}{s}\) unless \(s=1\). Participants suggest using the properties of prime factorization and the greatest common divisor (gcd) to establish these proofs, emphasizing the importance of contradiction and algebraic manipulation in the reasoning process.

PREREQUISITES
  • Understanding of rational numbers and their properties
  • Knowledge of prime factorization and irreducible fractions
  • Familiarity with the concept of greatest common divisor (gcd)
  • Basic proof techniques, including proof by contradiction
NEXT STEPS
  • Study the properties of gcd and its role in fraction simplification
  • Learn about prime factorization and its applications in number theory
  • Explore proof techniques in abstract algebra, particularly proof by contradiction
  • Investigate the implications of lowest terms in rational number comparisons
USEFUL FOR

Mathematicians, students of abstract algebra and number theory, and anyone interested in understanding the properties of rational numbers and proof techniques.

silvermane
Gold Member
Messages
113
Reaction score
0
I've been recently reading a book on abstract algebra and number theory, and I stumbled upon a problem that at first glance looked obvious, but I can't seem to figure out how to formally write the proof.

1.)So, let's say we have 4 integers, r,s,t,u, all greater than or equal to 1. Suppose \frac{r}{s} = \frac{t}{u} where both fractions are in lowest terms. Prove that r=t and s=u.

For this problem, I was thinking of solving it via contradiction but I can't seem to get there. I was thinking of using the fact that the gcd(r,s) = 1, and if the gcd(t,u)=1 that there would be a contradiction if they were not equal, but I feel like that's not enough. Any tips or hints would be greatly appreciated.

2.) Now, suppose we have r and s again (lowest terms), and we look at \frac{r}{s}. Prove that an integer N cannot equal \frac{r}{s} unless s = 1.

I was thinking of saying that we can write r as a product of primes:
r=p_{1}*p_{2}*...*p_{k}​

and then writing s as a product of primes, but primes that are all different from r's:
s=q_{1}*q_{2}*...*q_{k}​

Obviously through some algebraic manipulation, we see that \frac{r}{s} is also in lowest terms, and thus can't be an integer unless s=1, but I feel like I need more detail in this part of my proof.

Once again, thank you all in advance for your help and advice :)
 
Physics news on Phys.org
You're on the right track.

Let r = p_1 * p_2 * ... * p_a.

Let s = q_1 * q_2 * ... * q_b.

Let t = p'_1 * p'_2 * ... * p'_c.

Let u = q'_1 * q'_2 * ... * q'_d.

Cross multiply to get

(p_1 * p_2 * ... * p_k)(q'_1 * q'_2 * ... * q'_k) = (p'_1 * p'_2 * ... * p'_k)(q_1 * q_2 * ... * q_k) .

We know that p_i \neq q_k and p'_i \neq q'_k \ for all i and k.

Can you figure out the rest?
 
Raskolnikov said:
You're on the right track.

Let r = p_1 * p_2 * ... * p_a.

Let s = q_1 * q_2 * ... * q_b.

Let t = p'_1 * p'_2 * ... * p'_c.

Let u = q'_1 * q'_2 * ... * q'_d.

Cross multiply to get

(p_1 * p_2 * ... * p_k)(q'_1 * q'_2 * ... * q'_k) = (p'_1 * p'_2 * ... * p'_k)(q_1 * q_2 * ... * q_k) .

We know that p_i \neq q_k and p'_i \neq q'_k \ for all i and k.

Can you figure out the rest?


Yes, very much so! You've been more than helpful. I thought of doing it that way, but must have been distracted somehow. Thank you for all your help! :)
 
For the second one is it not enough to prove by contradiction?

Like suppose \frac{r}{s}=N then s divides r which leads to a contradiction ,unless s=1, given our choice of s and r namely gcd(r,s)=1.

Is this what you are doing ?
 
╔(σ_σ)╝ said:
For the second one is it not enough to prove by contradiction?

Like suppose \frac{r}{s}=N then s divides r which leads to a contradiction ,unless s=1, given our choice of s and r namely gcd(r,s)=1.

Is this what you are doing ?

Yes that's exactly what I've done. I just needed to know that I had to break them all into a product of irreducible primes to better understand what was going on. Thank you for your help :)
 
Question: A clock's minute hand has length 4 and its hour hand has length 3. What is the distance between the tips at the moment when it is increasing most rapidly?(Putnam Exam Question) Answer: Making assumption that both the hands moves at constant angular velocities, the answer is ## \sqrt{7} .## But don't you think this assumption is somewhat doubtful and wrong?

Similar threads

  • · Replies 5 ·
Replies
5
Views
3K
  • · Replies 13 ·
Replies
13
Views
4K
  • · Replies 4 ·
Replies
4
Views
2K
  • · Replies 1 ·
Replies
1
Views
1K
Replies
20
Views
3K
  • · Replies 1 ·
Replies
1
Views
2K
Replies
12
Views
2K
Replies
6
Views
2K
Replies
5
Views
2K
Replies
2
Views
1K